commom difference of an AP -4 , -4 , -4 ,............is...​

Answers

Answer 1

Answer:

0

Step-by-step explanation:

Common Difference = Difference between any two consecutive terms

= - 4 - (-4)

= - 4 + 4

= 0


Related Questions

DUE NOW PLEASE HELP!!!

Factor completely x2 − 10x + 25.

(x − 5)(x − 5)
(x + 5)(x + 5)
(x + 5)(x − 5)
(x − 25)(x − 1)

Answers

Answer:

(x - 5)(x - 5)

Step-by-step explanation:

[tex] {x}^{2} - 10x + 25 \: is \: the \: expansion \\ of \: {(x - 5)}^{2} \\ {(x - 5)}^{2} = (x - 5)(x - 5)[/tex]

The complete factorization of the quadratic expression x² - 10x + 25 is (x - 5)(x - 5). Hence the first option is the right choice.

How to factor a quadratic expression?

A quadratic expression of the form ax² + bx + c is factored by using the mid-term factorization method, which suggests that b should be broken in such two components that their product = ac. After this, we can factorize using the grouping method.

How to solve the given question?

In the question, we are asked to factor the quadratic expression x² - 10x + 25 completely.

Comparing x² - 10x + 25 to ax² + bx + c, we get a = 1, b = -10, and c = 25.

To factor the expression we will use the mid-term factorization method, and try to break b in such two numbers whose product = ac.

Now, ac = 1 * 25 = 25. b = -10, which can be broken as -5, and -5.

Therefore, we can write the given expression as:

x² - 10x + 25

= x² - 5x - 5x + 25, mid-term factorization

= x(x - 5) -5(x - 5), grouping

= (x - 5)(x - 5), grouping.

Therefore, the complete factorization of the quadratic expression x² - 10x + 25 is (x - 5)(x - 5). Hence the first option is the right choice.

Learn more about mid-term factorization at

https://brainly.com/question/25829061

#SPJ2

Keisha, Felipe, and Manuel sent a total of 100 text messages during the weekend. Keisha sent 8 more messages than Felipe. Manuel sent 2 times as many
messages as Felipe. How many messages did they each send?

Answers

Answer:

Felipe = 23 messages

Keisha = 31 messages

Manuel = 46 messages

Step-by-step explanation:

Keisha = K

Felipe = F

Manuel = M

=> There are a total of 100 messages.

=> K sent 8 +F => K = 8 + F

=> M sent 2 * F => M = 2F

=> F = F

=> 8 + F + 2F + F = 100

=> 8 + 4F = 100

=> 8 - 8 +4F = 100 -8

=> 4F = 92

=> 4F/4 = 92/4

=> F = 23

So, Felipe = 23 messages.

      Keisha = 8 + F = 8 + 23 = 31 messages.

      Manuel = 2F = 2* 23 = 46 messages.

46 + 31 + 23 = 77 + 23 = 100 messages.

So, the answer is correct.

andy is making floor plans for a tree house using a scale 1in to 2ft he wants to make the floor of the tree house have a length of 8ft. how many inches should he show for this distance on his floor plan

Answers

Answer:

Andy should represent the 8 feet long floor on the floor plan with a dimension of 4 inches

Step-by-step explanation:

The  scale of the tree house plan is given as 1 in. to 2 ft,

Therefore we have a scale of 1/2 in. of the floor plane is equivalent to 1 ft. in actual dimensions

Given that Andy wants the floor to make the tree house floor to have a length of 8 ft., let the dimension of the floor plan of the house floor be x, we have;

[tex]\dfrac{\frac{1}{2} \ inches \ plan }{1 \ feet \ actual} =\dfrac{x \ inches \ plan}{8 \ feet \ actual}[/tex]

[tex]x \ inches \ plan =\dfrac{\frac{1}{2} \ inches \ plan }{1 \ feet \ actual} \times 8 \ feet \ actual = 4 \ inches[/tex]

Therefore, Andy should represent the 8 feet long floor on the floor plan with a dimension of 4 inches.

Which of the c-values satisfy the following inequality? 2>c/3

Answers

Answer:

[tex]\Large \boxed{{c<6}}[/tex]

Step-by-step explanation:

2>c/3

Multiply both sides of the inequality by 3.

2(3)>c/3(3)

6>c

Switch sides.

c<6

●✴︎✴︎✴︎✴︎✴︎✴︎✴︎✴︎❀✴︎✴︎✴︎✴︎✴︎✴︎✴︎✴︎✴︎●

Hi my lil bunny!

❧⎯⎯⎯⎯⎯⎯⎯⎯⎯⎯⎯⎯⎯⎯⎯⎯⎯⎯⎯⎯⎯⎯⎯⎯⎯⎯⎯⎯⎯⎯⎯⎯⎯⎯⎯⎯⎯⎯☙

Let's solve your inequality step-by-step.

[tex]2 > \frac{c}{3}[/tex]

Step 1: Simplify both sides of the inequality.

[tex]2 > \frac{1}{3}c[/tex]

Step 2: Flip the equation.

[tex]\frac{1}{3}c < 2[/tex]

Step 3: Multiply both sides by 3.

[tex]3 * (\frac{1}{3} c) < ( 3) * (2)\\c < 6[/tex]

Answer : [tex]\boxed {c < 6}[/tex]

❧⎯⎯⎯⎯⎯⎯⎯⎯⎯⎯⎯⎯⎯⎯⎯⎯⎯⎯⎯⎯⎯⎯⎯⎯⎯⎯⎯⎯⎯⎯⎯⎯⎯⎯⎯⎯⎯⎯☙

●✴︎✴︎✴︎✴︎✴︎✴︎✴︎✴︎❀✴︎✴︎✴︎✴︎✴︎✴︎✴︎✴︎✴︎●

Have a great day/night!

❀*May*❀

Solve for p 9(p-4)=-18

Answers

Answer:

The answer is

p = 2

Step-by-step explanation:

9(p-4)=-18

First expand the terms in the bracket

that's

9p - 36 = - 18

Group like terms

Send the constants to the right side of the equation

That's

9p = - 18 + 36

9p = 18

Divide both sides by 9

That's

9p/9 = 18/9

We have the final answer as

p = 2

Hope this helps you

Answer:

[tex] \boxed{p = 2}[/tex]

Step-by-step explanation:

[tex] \mathsf{9(p - 4) = - 18}[/tex]

Distribute 9 through the parentheses

[tex] \mathsf{9p - 36 = - 18}[/tex]

Move constant to R.H.S and change it's sign

[tex] \mathsf{9p = - 18 + 36}[/tex]

Calculate

[tex] \mathsf{9p = 18}[/tex]

Divide both sides of the equation by 9

[tex] \mathsf{ \frac{9p}{9} = \frac{18}{9} }[/tex]

Calculate

[tex] \mathsf{p = 2}[/tex]

[tex] \mathcal{Hope \: I \: helped}[/tex]

[tex] \mathcal{Best \: regards}[/tex]

Jeania's parents have given her a interest-free loan of $100 to buy a new pair of running shoes She has to
pay back the loan with monthly payments of $20 each.
Write a function rule for the balance of the function (p), where p represents the number of
payments Jeania has made.

Answers

Answer:

The balance on the loan f(p) = $100 - $20 × p

Step-by-step explanation:

The parameters of the question are;

The loan amount = $100

The amount of monthly payment for the loan = $20

The function rule for the balance of the function f(p) where p is the number of payments is given as follows;

The balance on the loan, f(p) = The loan amount less the total amount paid

The total amount payment Jeania has made = Amount of monthly payment × Number of months paid, p

The total amount payment Jeania has made = $20 × p

∴ The balance on the loan, f(p) = $100 - $20 × p

Which gives;

f(p) = $100 - $20 × p.

Solve (s)(-3st)(-1/3)

Answers

Answer:

Step-by-step explanation

Plz help me this is probably easy but im just not seeing it plz help me ASAP

Answers

Answer: is attatched to the photo I sent

Step-by-step explanation:

I admit, this is tricky, just remember to think about it as where the shapes are the most similar. I'm not exactly sure how to explain it, but where it reflects like a mirror is the "copy"

The example I'm thinking of attached too, but these over lap so its diffucult

what is the range and domian of y=(x-4)

Answers

The answer is 8 because 4 over 4 up for y

the school bought a sandbox that measures 50 meters long 25 meters wide and 5 meters tall how many cubic meters if sand would you need to buy if each cubic meter of sand cost $1.50 how much money would it cost to fill the sandbox

Answers

Answer:

Cost of sandbox = $9,375

Step-by-step explanation:

Given:

Height of sandbox = 5 m

Length of sandbox = 50 m

Width of sandbox = 25 m

Cost of 1 cubic meter = $1.50

Find:

Cost of sandbox

Computation:

Volume of sandbox = (50)(25)(5)

Volume of sandbox = 6,250 m³

Cost of sandbox = 6,250 × $1.50

Cost of sandbox = $9,375

i think the answer. . .is the second one please correct me if i'm wrong

Answers

Answer: You are correct, it is the second option.

Step-by-step explanation:

Volume of a cylinder formula is: pi*r^2*h. The diameter is 6 and the radius is half the diameter so we get r=3. The height is 10 inches, so h=10. pi(3)^2(10) is the volume of the vase.

Volume of a sphere (marbles) formula is: 4/3*pi*r^3

The marbles have a diameter of 3 so 3/2=1.5. r=1.5.

The volume of the marbles is 8(4/3*pi*1.5^3).

Then you subtract the volume of the marbles from the volume of the vase to find the volume of the water in the vase.

pi(3)^2(10) - 8(4/3pi(1.5)^3)

Hope this helps. :)

Answer:

You are absolutely correct, second option is the correct answer.

Step-by-step explanation:

Diameter of vase = 6 inches

Therefore, radius r = 3 inches

Diameter of marbles = 3 inches

Radius of marbles = 1. 5 inches

Height of water h = 10 inches

Volume of water in the vase = Volume of vase - 8 times the volume of one marble

[tex] = \pi r^2h - 8\times \frac{4}{3} \pi r^3 \\\\

= \pi (3\: in) ^2(10\: in) - 8( \frac{4}{3} \pi (1.5\: in) ^3) \\\\[/tex]

Which answer choice identifies the relevant information in the problem? Sarah left the house at 12:15 p.M. To go to the store. She spent $42.20 on 2 books for her children and she spent $5.67 on a toys for her dog, Rover. Sarah arrived home at 1:00 p.M. How much did Sarah spend on each book? A. She spent $42.20 on 2 books. B. She spent $42.20 and $5.67. C. She left the house at 12:15 p.M. And arrived home at 1:00 p.M. D. You need to know how many children she has to solve the problem.

Answers

Answer:

Answer choices A, B and C identifies the relevant information in the problem

Step-by-step explanation:

Sarah left the house at 12:15 pm

She spent $42.20 on two books for her children

She spent $5.67 on a toy for her dog

Sarah arrived home at 1:00 pm

How much did Sarah spent on each book?

If she spent $42.20 on two books for her children,

Then, it means she has two children and the book cost $21.10 each

Answer choices A, B and C identifies the relevant information in the problem

Answer:

its A all the other one dont make sence sorry if im wrong but i got it right on my test

Step-by-step explanation:

Please please please please help

Answers

Answer:

[tex]x^2 +4x +3 [/tex]

Step-by-step explanation:

f(x)=x²-1

g(x)= x+2

f(g(x)) =f(x+2)

=(x+2)²-1

=x²+4x+4-1

=x²+4x+3

Between which two integers on a number line does -√120 lie on?

Answers

-11 can be written as - root 121 and -10 can be written as -100. Since 100 < 120 < 121, it lies between -11 and -10.

Answer:

-11 and -10

Step-by-step explanation:

● -√120 = -1 × √120

● -√120 = -1 × 2√30

● 30 is close to 25 so √30 is close to five but greater than it.

Multiplying 5 by -2 gives -10

Multipluing √30 by -2 gives you a number that is close to -10 but smaller than it.

So -√120 lies between -11 and -10

URGENT PLS HELP ASAP! THANK YOU :)​

Answers

Answer:

box 1 and box2 are correct.

write as an expression: a number that is equal to five less than b

Answers

Answer:

[tex]\huge\boxed{a = b-5}[/tex]

Step-by-step explanation:

Let the number be a

So, the given condition is:

a = b-5

Answer:

[tex]\Huge \boxed{a=b-5}[/tex]

Step-by-step explanation:

Let the number be [tex]a[/tex].

[tex]a[/tex] is equal to 5 less than [tex]b[/tex].

5 is subtracted from [tex]b[/tex].

Help, Answer ASAP; will give brainliest

Answers

Answer:

a = 2, b = 3

Step-by-step explanation:

The diagonals of a rectangle bisect each other, thus

5a² = 4a² + 4 ( subtract 4a² from both sides )

a² = 4 ( take the square root of both sides )

a = [tex]\sqrt{4}[/tex] = 2

Also

6b - 8 = 4b - 2 ( subtract 4b from both sides )

2b - 8 = - 2 ( add 8 to both sides )

2b = 6 ( divide both sides by 2 )

b = 3

Find the slope and Y-Intercept of the line. 6X plus 2Y equals -88

Answers

Answer:

That’s ez pz

Step-by-step explanation:

Answer:

The slope is -3 and the y intercept is -44

Step-by-step explanation:

6X+ 2Y= -88

The slope intercept form of a line is y= mx+b where m is the slope and b is the y intercept

Solve for y

6X-6x+ 2Y= -88-6x

2y = -6x-88

Divide by 2

y = -3x -44

The slope is -3 and the y intercept is -44

Determine the slope of a line which contains the points (2, 4) and (-6, 9). Write your answer in simplest form.

Answers

Answer:

-5/8

Step-by-step explanation:

(2,4) (-6.9)

m= y2-y1/x2-x1

= 9-4/-6-2

=5/-8

=-5/8

II
Initial Knowledge
This morning, Leila's car had 19.79 gallons of fuel. Now, 2.8 gallons are left. How much fuel did Leila use?

Answers

Answer:

[tex]19.79 - 2.8 = 16.99gallons[/tex]

En una fábrica de automóviles que trabaja las 24 horas se arman diariamente 24
automóviles tipo Sedan, 16 camionetas tipo SUV, 12 camionetas tipo VAN, 8
Camionetas tipo Pick-up y 2 automóviles deportivos.
Cl costo de producción y el precio de venta de cada vehículo es el siguiente:
Costo de
Vehículo
Precio de
Producción Venta
SEDAN
SEDAN
DEPORTIVO
$140,000 $185.000
SUV
$250,000
$320,000
VAN
$310,000
$400,000
PICK-UP
PICK-UP
$210,000
$285,000
VAN
DEPORTIVO
$400,000
$550,000
SUV
Cada año transcurrido, posterior a su fabricación, el precio de venta de los
vehículos disminuye una octava parte de su valor.
a suponiendo que en un día se vendan los vehículos en igual cantidad de los
que se fabricaron, como podrías calcular la ganancia?
b
Si la fábrica trabajara solo 12 horas, existe una forma de calcular cuántos
vehiculos se fabrican, ¿cuantos se fabricaron en este lapso? Sustenta tu
respuesta

Answers

Answer:

a. La ganancia es de $ 4,060,000.00

b. 31 vehículos

Step-by-step explanation:

(a) Los parámetros dados son;

El número de automóviles tipo sedán fabricados = 24

El número de camiones tipo SUV fabricados = 16

El número de camiones tipo VAN fabricados = 12

El número de camionetas pick-up fabricadas = 8

El número de autos deportivos fabricados = 2

La ganancia por la venta de autos tipo sedán = $ 185,000 - $ 140,000 = $ 40,000

La ganancia por la venta de camionetas tipo SUV = $ 320,000 - $ 250,000 = $ 70,000

La ganancia por la venta de camiones tipo VAN = $ 400,000 - $ 310,000 = $ 90,000

La ganancia por la venta de las camionetas pick-up = $ 285,000 - $ 210,000 = $ 75,000

La ganancia por la venta de los autos deportivos = $ 550,000 - $ 400,000 = $ 150,000

La ganancia = 24 * $ 40 000 + 16 * $ 70 000 + 12 * $ 90 000 + 8 * $ 75 000 + 2 * $ 150 000 = $ 4060 000

(b) Por lo que hay una tasa de producción constante, solo la mitad de los automóviles se producirán dentro del período de 12 horas

Por lo tanto, tu fabricado

12 autos sedán, 8 camionetas tipo SUV, 6 camionetas tipo VAN, 4 camionetas pick-up y 1 auto deportivo para hacer un total de 31 vehículos.

Rosemary walks each week for exercise. Let d represent the distance walked and h represent the number of hours spent walking Last weekwalked 18 miles in 6 hours This week d = 2.5h Which statement must be true?

Answers

THIS IS THE COMPLETE QUESTION BELOW;

Rosemary walks each week for exercise. Let d represent the distance walked and h represent the number of hours spent walking.

Last week: walked 18 miles in 6 hours

This week: d = 2.5h

Which statement must be true?

A.This week, she walked a greater distance.

B. Last week, she walked a greater distance

C. This week, she walked at a faster pace.

D. Last week, she walked at a faster pace

Answer

OPTION B is correct

B)Last week, she walked a greater distance

Step-by-step explanation:

We were told Rosemary walks each week for exercise.

From the question,

✓d represented the distance walked

✓h represent the number of hours spent walking.

A)Last week: she walked 18 miles in 6 hours

Then, if she walks 18 miles in 6 hours, we can be expressed as (18miles/6hour)

= 3 miles per hour

B)This week: d = 2.5h

This implies that she she walked 2.5 miles per hour this week since the distance is expressed in miles and time in hours.

So we can conclude that last week she walked 3 miles per hour which is more greater than 2.5 miles per hour which she walks this week.

Therefore, OPTION B is correct, (Last week, she walked a greater distance)

Answer:

It's b

Step-by-step explanation:

If 3sinA+4cosA=5 then find the value of cosA

Answers

Answer:

Cos A =1.4

Step-by-step explanation:

3sinA=5-4cosA; square both sides: 9sin^2(A)=25-40cosA+16cos^2(A);

9-9cos^2(A)=25-40cosA+16cos^2(A); 25cos^2(A)-40cosA+16=0=(5cosA-4)^2.

cosA=4/5 so sinA=3/5 (√(1-16/25) and sinA+cosA=7/5=1.4.

Samuel has $20 in his savings account before he makes a deposit of $160 after two weeks he withdraws $160 how did Samuel savings account balance change

Answers

Samuel’s saving account changed from 20 to 180 to back to 20

plz help ASAP! thank u

Answers

Answer: Choice B)

The relation is a function because there are no vertical lines that can be drawn on the graph that pass through more than one point.

This graph passes the vertical line test. Any input (x) leads to one and only one output (y). An example of a graph failing the vertical line test would be a graph that is a sideways parabola.

A certain mixture of paint contains 5 parts white paint for every 4 parts blue paint. If a can of paint contains 75 ounces of white paint, how many ounces of blue paint are in the can?

Answers

Answer:

i think 60 parts of blue paint are in the can.

Step-by-step explanation:

ratio should be equal in both cases

therefore,let blue part in second case be x(suppose)

5÷4=75÷x

by equating this we will be able to identify the value of x

and the value of x comes 60 which is the required answer....

hope this will help you..

i try my best to give correct answer..

if i am mistake, i am sorry for that...

A triangle has one side that lies along the line y=1/4x and another that lies along the line y=-1/4x. Which of the following points could be a vertex of the triangle?

Answers

Answer:

We know that our triangle has one side along the line:

y = (1/4)*x

And other side along the line:

y = -(1/4)*x.

Now, we want to find the vertex.

And we know that the vertex is the point where the two sides conect, so the vertex must be a common point of both lines.

Then we have:

y = (1/4)*x = -(1/4)*x

x = -x

The only solution to that equation is x = 0.

now we evaluate our lines in x = 0 and get:

y = (1/4)*0 = 0

y = -(1/4)*0 = 0

Then the lines intersect in the point (0, 0)

Then the vertex must be in the point (0, 0)

round your answer to the nearest hundredth. Find angle A=?

Answers

Answer:

A=48.81

Step-by-step explanation:

it is a right angle triangle find the hypotenuse c using Pythagorean theorem:

c²=a²+b²

c²=8²+7²

c=√64+49

c=10.63

sin A =opp/hyp

sin A=8/10.63

A=  48.81

another way :

tan A=opp/adj

tan A=8/7

A=48.81

The winning times (in seconds) in a speed-skating event for men can be represented by the formula T = 46.97 - 0.099x, where x represents the year, with x = 0 corresponding to 1920. (For example in 1992, x would be 1992 - 1920 = 72.) According to the formula, what was the winning time in 1997? Round to the nearest hundredth. * 1 point 40.34 sec 39.35 sec 3609.07 sec 41.33 sec

Answers

Answer:

39.35 sec

Step-by-step explanation:

Given that:

The winning time is represented by the function:

T = 46.97 - 0.099x

Where x = year ; x = 0 corresponding to 1920

According to the formula, what was the winning time in 1997?

first find the value of x;

x = 1997 - 1920 = 77 years

Nowing plugging the value of x in the function :

T = 46.97 - 0.099(77)

T = 46.97 - 7.623

T = 39.347 seconds

T = 39.35 s

Each day that a library book is kept past its due date a 30 dollar fee is charged at midnight which ordered pair is a viable solution if x represents the number of days that a library book is late and y represents the total fee

Answers

THIS IS THE COMPLETE QUESTION BELOW;

Each day that a library book is kept past its due date, a $0.30 fee is charged at midnight. Which ordered pair is a viable solution if x represents the number of days that a library book is late and y represents the total fee?

Answers:

(–3, –0.90)

(–2.5, –0.75)

(4.5, 1.35)

(8, 2.40)

Answer

(8, 2.40)

Step by step Explanation

✓We can denote the number of days library book is late = X,

✓We can denote the the total fee = Y.

We were told $0.30 fee is charged at midnight.

Then for lateness for just 1day,the charged fees= 1day ×

$0.30

For X number of days the charged fees= Xday ×$0.30

Therefore, total charge for lateness for X number of days late = Y.

Then can be expressed as

Y= 0.30 * X...............eqn(1)

We can now test the option one after the other

FIRST OPTION (-3, -0.9)

Here we should know the number of days cannot be negative so there is no need of testing in the equation (1)

SECOND OPTION (-2.5, -0.75)

Here we should know the number of days cannot be negative so there is no need of testing in the equation (1)

THIRD OPTION(4.5, 1.35)

here the number of days will definitely be a whole number not 4.5, it's either

charge for 4 days or 5 days.

FORTH OPTION (8, 2.40)

this should be correct because the number of days is whole number and not negative, then if we test it from our equation it satisfy the equation too

Y= 0.30 * X...............eqn(1)

Y= 0.30 * X

2.40= 0.30 * 8

2.40 = 2.40.

Therefore, (8, 2.40) is the answer

Other Questions
what is empowerment and radication please that is not from google Please answer this correctly without making mistakes Will mark as BRAINLIEST.......The position vector is given by vector r= 5t I cap + 2 t j cap + 2 k cap. Find it's velocity and acceleration at t=2s. Ms. Coulter was successful using a token reinforcement system with Jordan to decrease his inappropriate behaviors in her class. Mr. Johnson is now going to implement the same token system in his class in hopes that Jordan's behavior will improve. This is an example of what generalization technique? Group of answer choices What is the difference between sin^-1 and sin? Write the null and alternative hypotheses you would use to answer this question. Are Americans getting fatter? Researchers interested in this question take a random sample of 500 people and record an average weight of 190 pounds. Ten years ago, the average weight was 185 pounds. The before-trade domestic price of tomatoes in the United States is $500 per ton. The world price of tomatoes is $400 per ton. The U.S. is a price-taker in the tomatoes market. If trade in tomatoes is allowed, the United States:______ a) will experience increases in both consumer surplus and producer surplus. b) may become either an importer or an exporter of tomatoes, but this cannot be determined. c) will become an exporter of tomatoes. d) will become an importer of tomatoes. 11mg of cyanide per kilogram of body weight is lethal for 50% of domestic chickens. How many grams per kilogram of body weight is a lethal dose for 50% of domestic chickens? in a 10 team league, each teams play every other team exactly twice. find the total number of games played in the league El abuelo ____________ a la playa durante los veranos. a. iba b. iban c. ibas Please select the best answer from the choices provided A B C According to the United States Golf Association, the diameter of a golf ball should not be less than 42.67 millimeters. What is the estimate of this value rounded to the nearest tenth of a millimeter? Read the excerpt from "Flight into Yesterday." This leg was also the most dangerous of the entire trip. Howland, 2,600 miles to the east, was a tiny speck of land in the middle of the vast Pacific. It would take all of her skill as a pilot and Freds as a navigator to find the island. But danger did not worry her. She knew that flying carried risks. She faced them and accepted them. What mattered was setting herself a challenge and meeting it. What mattered was showing that women could do what men could do and encouraging other women to do what they were capable of. That was why she was flying over the Pacific, looking for a speck of land. Fame was never her goal. The important thing was to do what she had set out to do and to do it as well as she knew how. She had been that way all her life. Amelia Earhart, world-famous flier, was very much like the young girl who once lived in Atchison, Kansas. What best describes the authors purpose? (Economics) You have two friends who are pursuing different careers after high school this year. Danielle is interested in getting into the medical field and wants to be a doctor after eight years of schooling. Danny wants to be a barber after he earns a certificate after six months of barber school. 1)Predict: How will supply and demand and labor substitutes in these job markets affect their pay and likeliness to get a job?2)How will supply and demand within the job market affect my career choice? Gene is playing a game with a bag of marbles. 3 of the marbles are blue, 4 are green, and 7 are yellow. See below for awarded prizes. $2 green $0.5 yellow $4 blue What is the expected cost (or payout for Gene's game? The coefficient of 8.2 N is8216 please helppp ill give brainliest the question is attached below WILL CHOOSE BRAINLIEST Let Events A & B be described as follows: P(A) = watching a movie P(B) = going out to dinner The probability that a person will watch a movie is 62% and the probability of going out to dinner is 46%. The probability of watching a movie and going out to dinner is 28.52% Are watching a movie and going out to dinner independent events? Group of answer choices No, because the P(A)P(B) P(A and B). Yes, because the P(A)P(B) = P(A and B). No, because the P(A) + P(B) P(A and B). Yes, because the P(A) + P(B) is greater than 100%. What the settlement/ important places of the Rocky Mountains? Need answers asap. Will mark brailiest. If therefractive index of benzere is 2.419,what is the speed of light in benzene? michaela has h hair ties. michaela's sister has triple the number of hair ties that michaela has. choose the expression that shows how many hair bows michaela's sister has